Is it possible for $a^2bc+2, ab^2c+2, abc^2+2$ to all be perfect squares? [on hold] Announcing the arrival of Valued Associate #679: Cesar Manara Planned maintenance scheduled April 23, 2019 at 23:30 UTC (7:30pm US/Eastern)Prime Numbers And Perfect Squaresperfect squares possible?Sum of two numbers is equal to hcf +lcmShow that if $x,y,z$ are positive integers, then $(xy + 1)(yz + 1)(zx + 1)$ is a perfect square iff $xy +1, yz +1, zx+1$ are all perfect squares.Find all possible sum of digits of perfect squaresThere are two irreducible rational numbers with denominators 600 and 700. Find the minimal possible value of the denominator of their sum.An interesting number theory question for math contest training $x^2+x=2y^3$ (finding integer solutions)Prove that for all positive integers $n>1, (n^2+1)-n$ is not a perfect square.Prove that for all integers $r, s$ and $t$, that $gcd(gcd(r, s), t) = gcd(r, gcd(s, t))$.Prove that there are no positive integer solutions for $a,b,c,d$ if $a^2-b=c^2, b^2-a=d^2$

Universal covering space of the real projective line?

I can't produce songs

Mounting TV on a weird wall that has some material between the drywall and stud

Why is a lens darker than other ones when applying the same settings?

NERDTreeMenu Remapping

After Sam didn't return home in the end, were he and Al still friends?

What initially awakened the Balrog?

What is the chair depicted in Cesare Maccari's 1889 painting "Cicerone denuncia Catilina"?

Show current row "win streak"

How does light 'choose' between wave and particle behaviour?

Found this skink in my tomato plant bucket. Is he trapped? Or could he leave if he wanted?

Tannaka duality for semisimple groups

One-one communication

Nose gear failure in single prop aircraft: belly landing or nose-gear up landing?

What is the difference between CTSS and ITS?

Random body shuffle every night—can we still function?

The Nth Gryphon Number

retrieve food groups from food item list

In musical terms, what properties are varied by the human voice to produce different words / syllables?

How can a team of shapeshifters communicate?

Printing attributes of selection in ArcPy?

Test print coming out spongy

Did Mueller's report provide an evidentiary basis for the claim of Russian govt election interference via social media?

Flight departed from the gate 5 min before scheduled departure time. Refund options



Is it possible for $a^2bc+2, ab^2c+2, abc^2+2$ to all be perfect squares? [on hold]



Announcing the arrival of Valued Associate #679: Cesar Manara
Planned maintenance scheduled April 23, 2019 at 23:30 UTC (7:30pm US/Eastern)Prime Numbers And Perfect Squaresperfect squares possible?Sum of two numbers is equal to hcf +lcmShow that if $x,y,z$ are positive integers, then $(xy + 1)(yz + 1)(zx + 1)$ is a perfect square iff $xy +1, yz +1, zx+1$ are all perfect squares.Find all possible sum of digits of perfect squaresThere are two irreducible rational numbers with denominators 600 and 700. Find the minimal possible value of the denominator of their sum.An interesting number theory question for math contest training $x^2+x=2y^3$ (finding integer solutions)Prove that for all positive integers $n>1, (n^2+1)-n$ is not a perfect square.Prove that for all integers $r, s$ and $t$, that $gcd(gcd(r, s), t) = gcd(r, gcd(s, t))$.Prove that there are no positive integer solutions for $a,b,c,d$ if $a^2-b=c^2, b^2-a=d^2$










1












$begingroup$


This question in my opinion is challenging. I tried to put $gcd(a, b, c)=d$ and $a=dx, b=dy, c=dz$ but failed. Any help would be appreciated.










share|cite|improve this question











$endgroup$



put on hold as off-topic by user21820, Saad, Brahadeesh, Cesareo, Paul Frost Apr 17 at 15:45


This question appears to be off-topic. The users who voted to close gave this specific reason:


  • "This question is missing context or other details: Please provide additional context, which ideally explains why the question is relevant to you and our community. Some forms of context include: background and motivation, relevant definitions, source, possible strategies, your current progress, why the question is interesting or important, etc." – user21820, Saad, Brahadeesh, Cesareo, Paul Frost
If this question can be reworded to fit the rules in the help center, please edit the question.











  • 1




    $begingroup$
    Brute force says: Not if $a,b,c leq 3000$.
    $endgroup$
    – PierreCarre
    Apr 2 at 9:41















1












$begingroup$


This question in my opinion is challenging. I tried to put $gcd(a, b, c)=d$ and $a=dx, b=dy, c=dz$ but failed. Any help would be appreciated.










share|cite|improve this question











$endgroup$



put on hold as off-topic by user21820, Saad, Brahadeesh, Cesareo, Paul Frost Apr 17 at 15:45


This question appears to be off-topic. The users who voted to close gave this specific reason:


  • "This question is missing context or other details: Please provide additional context, which ideally explains why the question is relevant to you and our community. Some forms of context include: background and motivation, relevant definitions, source, possible strategies, your current progress, why the question is interesting or important, etc." – user21820, Saad, Brahadeesh, Cesareo, Paul Frost
If this question can be reworded to fit the rules in the help center, please edit the question.











  • 1




    $begingroup$
    Brute force says: Not if $a,b,c leq 3000$.
    $endgroup$
    – PierreCarre
    Apr 2 at 9:41













1












1








1


2



$begingroup$


This question in my opinion is challenging. I tried to put $gcd(a, b, c)=d$ and $a=dx, b=dy, c=dz$ but failed. Any help would be appreciated.










share|cite|improve this question











$endgroup$




This question in my opinion is challenging. I tried to put $gcd(a, b, c)=d$ and $a=dx, b=dy, c=dz$ but failed. Any help would be appreciated.







elementary-number-theory






share|cite|improve this question















share|cite|improve this question













share|cite|improve this question




share|cite|improve this question








edited Apr 2 at 19:22









Rick Almeida

1989




1989










asked Apr 2 at 8:35









user587054user587054

59111




59111




put on hold as off-topic by user21820, Saad, Brahadeesh, Cesareo, Paul Frost Apr 17 at 15:45


This question appears to be off-topic. The users who voted to close gave this specific reason:


  • "This question is missing context or other details: Please provide additional context, which ideally explains why the question is relevant to you and our community. Some forms of context include: background and motivation, relevant definitions, source, possible strategies, your current progress, why the question is interesting or important, etc." – user21820, Saad, Brahadeesh, Cesareo, Paul Frost
If this question can be reworded to fit the rules in the help center, please edit the question.







put on hold as off-topic by user21820, Saad, Brahadeesh, Cesareo, Paul Frost Apr 17 at 15:45


This question appears to be off-topic. The users who voted to close gave this specific reason:


  • "This question is missing context or other details: Please provide additional context, which ideally explains why the question is relevant to you and our community. Some forms of context include: background and motivation, relevant definitions, source, possible strategies, your current progress, why the question is interesting or important, etc." – user21820, Saad, Brahadeesh, Cesareo, Paul Frost
If this question can be reworded to fit the rules in the help center, please edit the question.







  • 1




    $begingroup$
    Brute force says: Not if $a,b,c leq 3000$.
    $endgroup$
    – PierreCarre
    Apr 2 at 9:41












  • 1




    $begingroup$
    Brute force says: Not if $a,b,c leq 3000$.
    $endgroup$
    – PierreCarre
    Apr 2 at 9:41







1




1




$begingroup$
Brute force says: Not if $a,b,c leq 3000$.
$endgroup$
– PierreCarre
Apr 2 at 9:41




$begingroup$
Brute force says: Not if $a,b,c leq 3000$.
$endgroup$
– PierreCarre
Apr 2 at 9:41










1 Answer
1






active

oldest

votes


















3












$begingroup$

First, observe that if any of $a, b, c$ is even, then one of the terms cannot be a square. WLOG take $a=2n$, then $4n^2bcequiv -2pmod4$ has no solution. So all of them must be odd.



Observe that only 1 is a quadratic residue mod 8.



$a^2bc+2equiv 1pmod8Longrightarrow a^2bcequiv (1)bc=bcequiv 7pmod8$



So we have the system



$$
abequiv 7pmod8quad acequiv 7pmod8quad bcequiv 7pmod8
$$



This implies $bequiv cpmod8$, hence from the last equation $b^2equiv 7pmod8$, but 7 is not a quadratic residue mode 8.



So there is no solution.






share|cite|improve this answer









$endgroup$



















    1 Answer
    1






    active

    oldest

    votes








    1 Answer
    1






    active

    oldest

    votes









    active

    oldest

    votes






    active

    oldest

    votes









    3












    $begingroup$

    First, observe that if any of $a, b, c$ is even, then one of the terms cannot be a square. WLOG take $a=2n$, then $4n^2bcequiv -2pmod4$ has no solution. So all of them must be odd.



    Observe that only 1 is a quadratic residue mod 8.



    $a^2bc+2equiv 1pmod8Longrightarrow a^2bcequiv (1)bc=bcequiv 7pmod8$



    So we have the system



    $$
    abequiv 7pmod8quad acequiv 7pmod8quad bcequiv 7pmod8
    $$



    This implies $bequiv cpmod8$, hence from the last equation $b^2equiv 7pmod8$, but 7 is not a quadratic residue mode 8.



    So there is no solution.






    share|cite|improve this answer









    $endgroup$

















      3












      $begingroup$

      First, observe that if any of $a, b, c$ is even, then one of the terms cannot be a square. WLOG take $a=2n$, then $4n^2bcequiv -2pmod4$ has no solution. So all of them must be odd.



      Observe that only 1 is a quadratic residue mod 8.



      $a^2bc+2equiv 1pmod8Longrightarrow a^2bcequiv (1)bc=bcequiv 7pmod8$



      So we have the system



      $$
      abequiv 7pmod8quad acequiv 7pmod8quad bcequiv 7pmod8
      $$



      This implies $bequiv cpmod8$, hence from the last equation $b^2equiv 7pmod8$, but 7 is not a quadratic residue mode 8.



      So there is no solution.






      share|cite|improve this answer









      $endgroup$















        3












        3








        3





        $begingroup$

        First, observe that if any of $a, b, c$ is even, then one of the terms cannot be a square. WLOG take $a=2n$, then $4n^2bcequiv -2pmod4$ has no solution. So all of them must be odd.



        Observe that only 1 is a quadratic residue mod 8.



        $a^2bc+2equiv 1pmod8Longrightarrow a^2bcequiv (1)bc=bcequiv 7pmod8$



        So we have the system



        $$
        abequiv 7pmod8quad acequiv 7pmod8quad bcequiv 7pmod8
        $$



        This implies $bequiv cpmod8$, hence from the last equation $b^2equiv 7pmod8$, but 7 is not a quadratic residue mode 8.



        So there is no solution.






        share|cite|improve this answer









        $endgroup$



        First, observe that if any of $a, b, c$ is even, then one of the terms cannot be a square. WLOG take $a=2n$, then $4n^2bcequiv -2pmod4$ has no solution. So all of them must be odd.



        Observe that only 1 is a quadratic residue mod 8.



        $a^2bc+2equiv 1pmod8Longrightarrow a^2bcequiv (1)bc=bcequiv 7pmod8$



        So we have the system



        $$
        abequiv 7pmod8quad acequiv 7pmod8quad bcequiv 7pmod8
        $$



        This implies $bequiv cpmod8$, hence from the last equation $b^2equiv 7pmod8$, but 7 is not a quadratic residue mode 8.



        So there is no solution.







        share|cite|improve this answer












        share|cite|improve this answer



        share|cite|improve this answer










        answered Apr 2 at 10:42









        Rick AlmeidaRick Almeida

        1989




        1989













            Popular posts from this blog

            Triangular numbers and gcdProving sum of a set is $0 pmod n$ if $n$ is odd, or $fracn2 pmod n$ if $n$ is even?Is greatest common divisor of two numbers really their smallest linear combination?GCD, LCM RelationshipProve a set of nonnegative integers with greatest common divisor 1 and closed under addition has all but finite many nonnegative integers.all pairs of a and b in an equation containing gcdTriangular Numbers Modulo $k$ - Hit All Values?Understanding the Existence and Uniqueness of the GCDGCD and LCM with logical symbolsThe greatest common divisor of two positive integers less than 100 is equal to 3. Their least common multiple is twelve times one of the integers.Suppose that for all integers $x$, $x|a$ and $x|b$ if and only if $x|c$. Then $c = gcd(a,b)$Which is the gcd of 2 numbers which are multiplied and the result is 600000?

            Ingelân Ynhâld Etymology | Geografy | Skiednis | Polityk en bestjoer | Ekonomy | Demografy | Kultuer | Klimaat | Sjoch ek | Keppelings om utens | Boarnen, noaten en referinsjes Navigaasjemenuwww.gov.ukOffisjele webside fan it regear fan it Feriene KeninkrykOffisjele webside fan it Britske FerkearsburoNederlânsktalige ynformaasje fan it Britske FerkearsburoOffisjele webside fan English Heritage, de organisaasje dy't him ynset foar it behâld fan it Ingelske kultuergoedYnwennertallen fan alle Britske stêden út 'e folkstelling fan 2011Notes en References, op dizze sideEngland

            Հադիս Բովանդակություն Անվանում և նշանակություն | Դասակարգում | Աղբյուրներ | Նավարկման ցանկ